2018년 유럽여학생수학올림피아드 6번문제

(a) 임의의 $0<t<\frac{1}{2}$인 실수 $t$에 대하여, 다음 성질을 만족시키는 양의 정수 $n$이 존재함을 보여라: $n$개의 양의 정수로 구성된 임의의 집합 $S$이 주어질 때, 그 집합 $S$에서 서로 다른 두 원소 $x$, $y$를 잘 뽑으면 적당한 음 아닌 정수 $m$에 대하여 \[ \lvert x-my\vert \le ty\]가 성립한다.

(b) 임의의 $0<t<\frac{1}{2}$인 실수 $t$에 대하여 적당한 양의 정수로 구성된 무한 집합 $S$를 잘 고르면, 그 집합 $S$에서 서로 다른 두 임의의 원소 $x$, $y$를 잘 뽑으면 임의의 양의 정수 $m$에 대하여 \[\lvert x-my\rvert >ty\]인가?

 

GD Star Rating
loading...
이 글은 정수 카테고리에 분류되었고 mo님에 의해 작성되었습니다. 고유주소 북마크.